1980 AHSME Problems/Problem 28

Revision as of 19:38, 30 October 2021 by Angelalz (talk | contribs) (Solution)

Problem

The polynomial $x^{2n}+1+(x+1)^{2n}$ is not divisible by $x^2+x+1$ if $n$ equals

$\text{(A)} \ 17 \qquad  \text{(B)} \ 20 \qquad  \text{(C)} \ 21 \qquad  \text{(D)} \ 64 \qquad  \text{(E)} \ 65$

Solution

Let $h(x)=x^2+x+1$.

Then we have \[(x+1)^2n = (x^2+2x+1)^n = (h(x)+x)^n = g(x) \cdot h(x) + x^n,\] where $g(x)$ is $h^{n-1}(x) + nh^{n-2}(x) \cdot x + ... + x^{n-1}$ (after expanding $(h(x)+x)^n$ according to the Binomial Theorem.

Notice that $$ (Error compiling LaTeX. Unknown error_msg)x^2n = x^2n+x^{2n-1}+x^{2n-2}+...x

          -x^{2n-1}-x^{2n-2}-x^{2n-3}
        +...$$ (Error compiling LaTeX. Unknown error_msg)x^n = x^n+x^{n-1}+x^{n-2}
        -x^{n-1}-x^{n-2}-x^{n-3}
 +....$Therefore, the left term from$x^2n$is$x^{(2n-3u)}$the left term from$x^n$is$x^{(n-3v)}$, 

If divisible by h(x), we need 2n-3u=1 and n-3v=2 or

                             2n-3u=2 and n-3v=1

The solution will be n=1 or 2 mod(3). Therefore n=21 is impossible

~~Wei

See also

1980 AHSME (ProblemsAnswer KeyResources)
Preceded by
Problem 27
Followed by
Problem 29
1 2 3 4 5 6 7 8 9 10 11 12 13 14 15 16 17 18 19 20 21 22 23 24 25 26 27 28 29 30
All AHSME Problems and Solutions

The problems on this page are copyrighted by the Mathematical Association of America's American Mathematics Competitions. AMC logo.png